Join our FREE webinar on May 1st to learn about managing anxiety.

G
Topic
First Poster
Last Poster
k a April Highlights and 2025 AoPS Online Class Information
jlacosta   0
Apr 2, 2025
Spring is in full swing and summer is right around the corner, what are your plans? At AoPS Online our schedule has new classes starting now through July, so be sure to keep your skills sharp and be prepared for the Fall school year! Check out the schedule of upcoming classes below.

WOOT early bird pricing is in effect, don’t miss out! If you took MathWOOT Level 2 last year, no worries, it is all new problems this year! Our Worldwide Online Olympiad Training program is for high school level competitors. AoPS designed these courses to help our top students get the deep focus they need to succeed in their specific competition goals. Check out the details at this link for all our WOOT programs in math, computer science, chemistry, and physics.

Looking for summer camps in math and language arts? Be sure to check out the video-based summer camps offered at the Virtual Campus that are 2- to 4-weeks in duration. There are middle and high school competition math camps as well as Math Beasts camps that review key topics coupled with fun explorations covering areas such as graph theory (Math Beasts Camp 6), cryptography (Math Beasts Camp 7-8), and topology (Math Beasts Camp 8-9)!

Be sure to mark your calendars for the following events:
[list][*]April 3rd (Webinar), 4pm PT/7:00pm ET, Learning with AoPS: Perspectives from a Parent, Math Camp Instructor, and University Professor
[*]April 8th (Math Jam), 4:30pm PT/7:30pm ET, 2025 MATHCOUNTS State Discussion
April 9th (Webinar), 4:00pm PT/7:00pm ET, Learn about Video-based Summer Camps at the Virtual Campus
[*]April 10th (Math Jam), 4:30pm PT/7:30pm ET, 2025 MathILy and MathILy-Er Math Jam: Multibackwards Numbers
[*]April 22nd (Webinar), 4:00pm PT/7:00pm ET, Competitive Programming at AoPS (USACO).[/list]
Our full course list for upcoming classes is below:
All classes run 7:30pm-8:45pm ET/4:30pm - 5:45pm PT unless otherwise noted.

Introductory: Grades 5-10

Prealgebra 1 Self-Paced

Prealgebra 1
Sunday, Apr 13 - Aug 10
Tuesday, May 13 - Aug 26
Thursday, May 29 - Sep 11
Sunday, Jun 15 - Oct 12
Monday, Jun 30 - Oct 20
Wednesday, Jul 16 - Oct 29

Prealgebra 2 Self-Paced

Prealgebra 2
Sunday, Apr 13 - Aug 10
Wednesday, May 7 - Aug 20
Monday, Jun 2 - Sep 22
Sunday, Jun 29 - Oct 26
Friday, Jul 25 - Nov 21

Introduction to Algebra A Self-Paced

Introduction to Algebra A
Monday, Apr 7 - Jul 28
Sunday, May 11 - Sep 14 (1:00 - 2:30 pm ET/10:00 - 11:30 am PT)
Wednesday, May 14 - Aug 27
Friday, May 30 - Sep 26
Monday, Jun 2 - Sep 22
Sunday, Jun 15 - Oct 12
Thursday, Jun 26 - Oct 9
Tuesday, Jul 15 - Oct 28

Introduction to Counting & Probability Self-Paced

Introduction to Counting & Probability
Wednesday, Apr 16 - Jul 2
Thursday, May 15 - Jul 31
Sunday, Jun 1 - Aug 24
Thursday, Jun 12 - Aug 28
Wednesday, Jul 9 - Sep 24
Sunday, Jul 27 - Oct 19

Introduction to Number Theory
Thursday, Apr 17 - Jul 3
Friday, May 9 - Aug 1
Wednesday, May 21 - Aug 6
Monday, Jun 9 - Aug 25
Sunday, Jun 15 - Sep 14
Tuesday, Jul 15 - Sep 30

Introduction to Algebra B Self-Paced

Introduction to Algebra B
Wednesday, Apr 16 - Jul 30
Tuesday, May 6 - Aug 19
Wednesday, Jun 4 - Sep 17
Sunday, Jun 22 - Oct 19
Friday, Jul 18 - Nov 14

Introduction to Geometry
Wednesday, Apr 23 - Oct 1
Sunday, May 11 - Nov 9
Tuesday, May 20 - Oct 28
Monday, Jun 16 - Dec 8
Friday, Jun 20 - Jan 9
Sunday, Jun 29 - Jan 11
Monday, Jul 14 - Jan 19

Intermediate: Grades 8-12

Intermediate Algebra
Monday, Apr 21 - Oct 13
Sunday, Jun 1 - Nov 23
Tuesday, Jun 10 - Nov 18
Wednesday, Jun 25 - Dec 10
Sunday, Jul 13 - Jan 18
Thursday, Jul 24 - Jan 22

Intermediate Counting & Probability
Wednesday, May 21 - Sep 17
Sunday, Jun 22 - Nov 2

Intermediate Number Theory
Friday, Apr 11 - Jun 27
Sunday, Jun 1 - Aug 24
Wednesday, Jun 18 - Sep 3

Precalculus
Wednesday, Apr 9 - Sep 3
Friday, May 16 - Oct 24
Sunday, Jun 1 - Nov 9
Monday, Jun 30 - Dec 8

Advanced: Grades 9-12

Olympiad Geometry
Tuesday, Jun 10 - Aug 26

Calculus
Tuesday, May 27 - Nov 11
Wednesday, Jun 25 - Dec 17

Group Theory
Thursday, Jun 12 - Sep 11

Contest Preparation: Grades 6-12

MATHCOUNTS/AMC 8 Basics
Wednesday, Apr 16 - Jul 2
Friday, May 23 - Aug 15
Monday, Jun 2 - Aug 18
Thursday, Jun 12 - Aug 28
Sunday, Jun 22 - Sep 21
Tues & Thurs, Jul 8 - Aug 14 (meets twice a week!)

MATHCOUNTS/AMC 8 Advanced
Friday, Apr 11 - Jun 27
Sunday, May 11 - Aug 10
Tuesday, May 27 - Aug 12
Wednesday, Jun 11 - Aug 27
Sunday, Jun 22 - Sep 21
Tues & Thurs, Jul 8 - Aug 14 (meets twice a week!)

AMC 10 Problem Series
Friday, May 9 - Aug 1
Sunday, Jun 1 - Aug 24
Thursday, Jun 12 - Aug 28
Tuesday, Jun 17 - Sep 2
Sunday, Jun 22 - Sep 21 (1:00 - 2:30 pm ET/10:00 - 11:30 am PT)
Monday, Jun 23 - Sep 15
Tues & Thurs, Jul 8 - Aug 14 (meets twice a week!)

AMC 10 Final Fives
Sunday, May 11 - Jun 8
Tuesday, May 27 - Jun 17
Monday, Jun 30 - Jul 21

AMC 12 Problem Series
Tuesday, May 27 - Aug 12
Thursday, Jun 12 - Aug 28
Sunday, Jun 22 - Sep 21
Wednesday, Aug 6 - Oct 22

AMC 12 Final Fives
Sunday, May 18 - Jun 15

F=ma Problem Series
Wednesday, Jun 11 - Aug 27

WOOT Programs
Visit the pages linked for full schedule details for each of these programs!


MathWOOT Level 1
MathWOOT Level 2
ChemWOOT
CodeWOOT
PhysicsWOOT

Programming

Introduction to Programming with Python
Thursday, May 22 - Aug 7
Sunday, Jun 15 - Sep 14 (1:00 - 2:30 pm ET/10:00 - 11:30 am PT)
Tuesday, Jun 17 - Sep 2
Monday, Jun 30 - Sep 22

Intermediate Programming with Python
Sunday, Jun 1 - Aug 24
Monday, Jun 30 - Sep 22

USACO Bronze Problem Series
Tuesday, May 13 - Jul 29
Sunday, Jun 22 - Sep 1

Physics

Introduction to Physics
Wednesday, May 21 - Aug 6
Sunday, Jun 15 - Sep 14
Monday, Jun 23 - Sep 15

Physics 1: Mechanics
Thursday, May 22 - Oct 30
Monday, Jun 23 - Dec 15

Relativity
Sat & Sun, Apr 26 - Apr 27 (4:00 - 7:00 pm ET/1:00 - 4:00pm PT)
Mon, Tue, Wed & Thurs, Jun 23 - Jun 26 (meets every day of the week!)
0 replies
jlacosta
Apr 2, 2025
0 replies
k i Adding contests to the Contest Collections
dcouchman   1
N Apr 5, 2023 by v_Enhance
Want to help AoPS remain a valuable Olympiad resource? Help us add contests to AoPS's Contest Collections.

Find instructions and a list of contests to add here: https://artofproblemsolving.com/community/c40244h1064480_contests_to_add
1 reply
dcouchman
Sep 9, 2019
v_Enhance
Apr 5, 2023
k i Zero tolerance
ZetaX   49
N May 4, 2019 by NoDealsHere
Source: Use your common sense! (enough is enough)
Some users don't want to learn, some other simply ignore advises.
But please follow the following guideline:


To make it short: ALWAYS USE YOUR COMMON SENSE IF POSTING!
If you don't have common sense, don't post.


More specifically:

For new threads:


a) Good, meaningful title:
The title has to say what the problem is about in best way possible.
If that title occured already, it's definitely bad. And contest names aren't good either.
That's in fact a requirement for being able to search old problems.

Examples:
Bad titles:
- "Hard"/"Medium"/"Easy" (if you find it so cool how hard/easy it is, tell it in the post and use a title that tells us the problem)
- "Number Theory" (hey guy, guess why this forum's named that way¿ and is it the only such problem on earth¿)
- "Fibonacci" (there are millions of Fibonacci problems out there, all posted and named the same...)
- "Chinese TST 2003" (does this say anything about the problem¿)
Good titles:
- "On divisors of a³+2b³+4c³-6abc"
- "Number of solutions to x²+y²=6z²"
- "Fibonacci numbers are never squares"


b) Use search function:
Before posting a "new" problem spend at least two, better five, minutes to look if this problem was posted before. If it was, don't repost it. If you have anything important to say on topic, post it in one of the older threads.
If the thread is locked cause of this, use search function.

Update (by Amir Hossein). The best way to search for two keywords in AoPS is to input
[code]+"first keyword" +"second keyword"[/code]
so that any post containing both strings "first word" and "second form".


c) Good problem statement:
Some recent really bad post was:
[quote]$lim_{n\to 1}^{+\infty}\frac{1}{n}-lnn$[/quote]
It contains no question and no answer.
If you do this, too, you are on the best way to get your thread deleted. Write everything clearly, define where your variables come from (and define the "natural" numbers if used). Additionally read your post at least twice before submitting. After you sent it, read it again and use the Edit-Button if necessary to correct errors.


For answers to already existing threads:


d) Of any interest and with content:
Don't post things that are more trivial than completely obvious. For example, if the question is to solve $x^{3}+y^{3}=z^{3}$, do not answer with "$x=y=z=0$ is a solution" only. Either you post any kind of proof or at least something unexpected (like "$x=1337, y=481, z=42$ is the smallest solution). Someone that does not see that $x=y=z=0$ is a solution of the above without your post is completely wrong here, this is an IMO-level forum.
Similar, posting "I have solved this problem" but not posting anything else is not welcome; it even looks that you just want to show off what a genius you are.

e) Well written and checked answers:
Like c) for new threads, check your solutions at least twice for mistakes. And after sending, read it again and use the Edit-Button if necessary to correct errors.



To repeat it: ALWAYS USE YOUR COMMON SENSE IF POSTING!


Everything definitely out of range of common sense will be locked or deleted (exept for new users having less than about 42 posts, they are newbies and need/get some time to learn).

The above rules will be applied from next monday (5. march of 2007).
Feel free to discuss on this here.
49 replies
ZetaX
Feb 27, 2007
NoDealsHere
May 4, 2019
Add a digit to obtain a new perfect square
Lukaluce   2
N 4 minutes ago by TopGbulliedU
Source: 2024 Junior Macedonian Mathematical Olympiad P4
Let $a_1, a_2, ..., a_n$ be a sequence of perfect squares such that $a_{i + 1}$ can be obtained by concatenating a digit to the right of $a_i$. Determine all such sequences that are of maximum length.

Proposed by Ilija Jovčeski
2 replies
Lukaluce
Apr 14, 2025
TopGbulliedU
4 minutes ago
D1025 : Can you do that?
Dattier   2
N 7 minutes ago by CerealCipher
Source: les dattes à Dattier
Let $x_{n+1}=x_n^3$ and $x_0=3$.

Can you calculate $\sum\limits_{i=1}^{2^{2025}} x_i \mod 10^{30}$?
2 replies
Dattier
Yesterday at 8:24 PM
CerealCipher
7 minutes ago
Simple inequality
sqing   7
N 13 minutes ago by sqing
Source: Daniel Sitaru
Let $a,b,c>0$ . Prove that$$\frac{a^3}{b^3}+\frac{b^3}{c^3}+\frac{c^3}{a^3}+9>\frac{3}{2}\left(\frac{a^2}{b^2}+\frac{b^2}{c^2}+\frac{c^2}{a^2}+
\frac{a}{b}+\frac{b}{c}+\frac{c}{a}\right)$$
7 replies
sqing
Feb 10, 2017
sqing
13 minutes ago
Vector Vortex
steven_zhang123   1
N 13 minutes ago by Mathzeus1024
Source: NS Issue 1 P3 (2014.4)
Let $v_{1}, v_{2}, \cdots, v_{n}$ be $n$ unit vectors on a plane, where $n$ is an odd number. Prove that there exist $\varepsilon _i\in \left \{ -1,1 \right \} $ for $i=1,2,\cdots,n$ such that $\left | \sum_{i=1}^{n} \varepsilon_i v_i \right | \le 1.$
1 reply
steven_zhang123
Feb 15, 2025
Mathzeus1024
13 minutes ago
China Northern MO 2009 p4 CNMO
parkjungmin   0
13 minutes ago
Source: China Northern MO 2009 p4 CNMO P4
The problem is too difficult.
0 replies
parkjungmin
13 minutes ago
0 replies
The Appetizer of Iran NT2023
alinazarboland   6
N 27 minutes ago by A22-
Source: Iran MO 3rd round 2023 NT exam , P1
Find all integers $n > 4$ st for every two subsets $A,B$ of $\{0,1,....,n-1\}$ , there exists a polynomial $f$ with integer coefficients st either $f(A) = B$ or $f(B) = A$ where the equations are considered mod n.
We say two subsets are equal mod n if they produce the same set of reminders mod n. and the set $f(X)$ is the set of reminders of $f(x)$ where $x \in X$ mod n.
6 replies
alinazarboland
Aug 17, 2023
A22-
27 minutes ago
Do not try to case bash lol
ItzsleepyXD   1
N 27 minutes ago by Haris1
Source: Own , Mock Thailand Mathematic Olympiad P3
Let $n,d\geqslant 6$ be a positive integer such that $d\mid 6^{n!}+1$ .
Prove that $d>2n+6$ .
1 reply
ItzsleepyXD
an hour ago
Haris1
27 minutes ago
Rutthee on some APMO style
ItzsleepyXD   0
29 minutes ago
Source: Own , Mock Thailand Mathematic Olympiad P10 (Not Rutthee problem , Idk what to name a sequence)
Let $a_0,a_1,\dots$ be Rutthee sequence if $a_0$ be a positive integer and
$$a_{i}\in\left\{3a_{i-1}+2,\frac{2a_{i-1}+1}{a_{i-1}+2},\frac{a_{i-1}}{2a_{i-1}+3}\right\}$$for all $i \in \mathbb{Z^+}$ and there are some positive integer $n$ sastisfied $a_n\in\{2025,2568\}$
Is it possible that there are Rutthee sequence such that there exist positive integer $m\neq n$ such that $a_m=2025$ and $a_n=2568$ also find all possible value of $a_0$ in Rutthee sequence
0 replies
ItzsleepyXD
29 minutes ago
0 replies
3 var inequality
sqing   0
33 minutes ago
Source: Own
Let $ a,b,c>0 ,\frac{a}{b} +\frac{b}{c} +\frac{c}{a} \leq 2\left( \frac{1}{a}+\frac{1}{b}+\frac{1}{c}\right). $ Prove that
$$a+b+c+2\geq abc$$Let $ a,b,c>0 , a^3+b^3+c^3\leq 2(ab+bc+ca). $ Prove that
$$a+b+c+2\geq abc$$
0 replies
sqing
33 minutes ago
0 replies
Do not try to bash on beautiful geometry
ItzsleepyXD   0
33 minutes ago
Source: Own , Mock Thailand Mathematic Olympiad P9
Let $ABC$be triangle with point $D,E$ and $F$ on $BC,AB,CA$
such that $BE=CF$ and $E,F$ are on the same side of $BC$
Let $M$ be midpoint of segment $BC$ and $N$ be midpoint of segment $EF$
Let $G$ be intersection of $BF$ with $CE$ and $\dfrac{BD}{DC}=\dfrac{AC}{AB}$
Prove that $MN\parallel DG$
0 replies
ItzsleepyXD
33 minutes ago
0 replies
already well-known, but yet strangely difficult
Valentin Vornicu   37
N 35 minutes ago by cursed_tangent1434
Source: Romanian ROM TST 2004, problem 6
Let $a,b$ be two positive integers, such that $ab\neq 1$. Find all the integer values that $f(a,b)$ can take, where \[ f(a,b) = \frac { a^2+ab+b^2} { ab- 1} . \]
37 replies
Valentin Vornicu
May 1, 2004
cursed_tangent1434
35 minutes ago
1 line solution to Inequality
ItzsleepyXD   0
36 minutes ago
Source: Own , Mock Thailand Mathematic Olympiad P8
Let $x_1,x_2,\dots,x_n$ be positive real integer such that $x_1^2+x_2^2+\cdots+x_n^2=2$ Prove that
$$\sum_{i=1}^{n}\frac{1}{x_i^3(x_{i-1}+x_{i+1})}\geqslant \left(\sum_{i=1}^{n}\frac{x_i}{x_{i-1}+x_{i+1}}\right)^3$$such that $x_{n+1}=x_1$ and $x_0=x_n$
0 replies
ItzsleepyXD
36 minutes ago
0 replies
Invariant board combi style
ItzsleepyXD   0
38 minutes ago
Source: Own , Mock Thailand Mathematic Olympiad P7
Oh write $2025^{2025^{2025}}$ real number on the board such that each number is more than $2025^{2025}$ .
Oh erase 2 number $x,y$ on the board and write $\frac{xy-2025}{x+y-90}$ .
Prove that the last number will always be the same regardless the order of number that Oh pick .
0 replies
1 viewing
ItzsleepyXD
38 minutes ago
0 replies
finite solutions (CGMO2009/1)
earldbest   6
N 39 minutes ago by Namisgood
Source: China Girls Mathematical Olympiad 2009, Problem 1
Show that there are only finitely many triples $ (x,y,z)$ of positive integers satisfying the equation $ abc=2009(a+b+c).$
6 replies
earldbest
Aug 18, 2009
Namisgood
39 minutes ago
Strange Conditional Sequence
MarkBcc168   21
N Mar 4, 2025 by pie854
Source: APMO 2019 P2
Let $m$ be a fixed positive integer. The infinite sequence $\{a_n\}_{n\geq 1}$ is defined in the following way: $a_1$ is a positive integer, and for every integer $n\geq 1$ we have
$$a_{n+1} = \begin{cases}a_n^2+2^m & \text{if } a_n< 2^m \\ a_n/2 &\text{if } a_n\geq 2^m\end{cases}$$For each $m$, determine all possible values of $a_1$ such that every term in the sequence is an integer.
21 replies
MarkBcc168
Jun 11, 2019
pie854
Mar 4, 2025
Strange Conditional Sequence
G H J
G H BBookmark kLocked kLocked NReply
Source: APMO 2019 P2
The post below has been deleted. Click to close.
This post has been deleted. Click here to see post.
MarkBcc168
1595 posts
#1 • 2 Y
Y by Adventure10, Mango247
Let $m$ be a fixed positive integer. The infinite sequence $\{a_n\}_{n\geq 1}$ is defined in the following way: $a_1$ is a positive integer, and for every integer $n\geq 1$ we have
$$a_{n+1} = \begin{cases}a_n^2+2^m & \text{if } a_n< 2^m \\ a_n/2 &\text{if } a_n\geq 2^m\end{cases}$$For each $m$, determine all possible values of $a_1$ such that every term in the sequence is an integer.
This post has been edited 1 time. Last edited by MarkBcc168, Jun 12, 2019, 2:42 PM
Z K Y
The post below has been deleted. Click to close.
This post has been deleted. Click here to see post.
MarkBcc168
1595 posts
#2 • 2 Y
Y by Bunrong123, Adventure10
Solution
Z K Y
The post below has been deleted. Click to close.
This post has been deleted. Click here to see post.
Anaskudsi
112 posts
#3 • 10 Y
Y by AbodeMokayed, ks_789, karitoshi, ArMath, ILOVEMYFAMILY, Muaaz.SY, MatBoy-123, SADAT, Adventure10, Mango247
$\textcolor{blue}{\textbf{-}}$ The only solution is $m=2$ and $a_1=2^r$ for some positive integer $r$.
$\textcolor{red}{\textbf{\underline{Claim.1:}}}$ There is no $i\ge 1$ such that $a_i$ is odd.
$\textcolor{blue}{\textbf{The proof:}}  $
Assume that there exist $i\ge 1$ such that $a_i=s$ is odd.
If $s\ge 2^m$ then $a_{i+1}=\frac{s}{2}$ and this is not integer.
If $s<2^m$ then $a_{i+1}=s^2+2^m \Longrightarrow a_{i+2}=\frac{s^2+2^m}{2}$ and this is not integer.
Contradiction, so There is no $i\ge 1$ such that $a_i$ is odd.

$\textcolor{red}{\textbf{\underline{Claim.2:}}} $ All the odd parts of the terms of the sequence $(a_n) _{n \ge 1}$ are less than $2^{m-1}$.
$\textcolor{blue}{\textbf{The proof:}} $
Assume that there exist $a_i=2^r s$ such that $gcd(s,2)=1$ and $s \ge 2^{m-1}$ then.
$a_{i+1}=2^{r-1} s, \cdots, a_{i+r-1}=2s$ but $2s\ge 2^m$ so $a_{i+r}=s$ and this is a contradiction with claim.1 so always $s<2^{m-1}$.

Now let's discuss the following cases:
$\textcolor{red}{\textbf{-}}$ When $m=1$, let $a_1=2^r s$, we know by using claim.2 that $s<1$ and this is a contradiction. So there is no possible values of $a_1$.
$\textcolor{red}{\textbf{-}}$ When $m=2$, let $a_1=2^r s$, we know by using claim.2 that $s<2$ so $s=1$.
$a_1=2^r \Longrightarrow a_2=2^{r-1}, \cdots, a_r=2 \Longrightarrow a_{r+1}=8 \Longrightarrow a_{r+2}=4 \Longrightarrow a_{r+3}=2$ .
So $a_{r+3k}=2,a_{r+3k+1}=8,a_{r+3k+2}=4$ for every integer $k \ge 0$, so all the terms of the sequence $(a_n)_{n \ge 1}$ are integers when $m=2$ and $a_1=2^r$ for some positive integer $r$.
$\textcolor{red}{\textbf{-}}$ When $m \ge 3$, let $a_i=2^r s$, assume that $a_i<2^m$ otherwise we can divide by $2$ so many times to achieve to a term that is less than $2^m$. So $a_{i+1}=2^{2r}s^2+2^m$:

1- If $2r>m$ then $a_{i+1}=2^m(2^{2r-m}s^2+1) $ and if $m>2r$ then $a_{i+1}=2^{2r}(s^2+2^{m-2r})$,but $2^{2r-m}s^2+1>s$ and $ s^2+2^{m-2r}>s$. So the odd part is increasing.
2- If $m=2r$ then $a_{i+1}=2^m(s^2+1)$, here we know that $4 \nmid s^2+1$ so $\frac{s^2+1}{2}$ is the odd part of $a_{i+1}$, but we know by using $AM-GM$ inequality that $\frac{s^2+1}{2} \ge s$ and the equality holds if and only if $s=1$, but if $s=1$ then $a_{i+1}=2^{m+1} \Rightarrow a_{i+3}=2^{m-1} \Rightarrow a_{i+4}=2^m(2^{m-2}+1)$, but $2^{m-2}+1>s=1$ and if $s \neq 1$ then $\frac{s^2+1}{2} > s$, so the odd part is increasing as well. So this is a contradiction with claim.2 because the odd part is increasing but it is always less than $2^{m-1}$. So there is no possible values of $a_1$.
$\blacksquare$
This post has been edited 1 time. Last edited by Anaskudsi, Jun 11, 2019, 5:21 PM
Z K Y
The post below has been deleted. Click to close.
This post has been deleted. Click here to see post.
rocketscience
466 posts
#4 • 3 Y
Y by alex_g, Adventure10, Mango247
Should the problem statement read "if $a_n < 2^m$" and "if $a_n \ge 2^m$"?
Z K Y
The post below has been deleted. Click to close.
This post has been deleted. Click here to see post.
MarkBcc168
1595 posts
#5 • 2 Y
Y by Adventure10, Mango247
rocketscience wrote:
Should the problem statement read "if $a_n < 2^m$" and "if $a_n \ge 2^m$"?

Fixed, thanks.
Z K Y
The post below has been deleted. Click to close.
This post has been deleted. Click here to see post.
plagueis
157 posts
#6 • 7 Y
Y by e_plus_pi, TLP.39, Aryan-23, Justpassingby, sabkx, Adventure10, Deadline
This problem was proposed by me. The solutions I sent are exactly the same as those in the replies, but in different words, so I won't put them here.
Z K Y
The post below has been deleted. Click to close.
This post has been deleted. Click here to see post.
Ali3085
214 posts
#7 • 2 Y
Y by Adventure10, Mango247
claim(1): if $a_i < 2^m$ then $a_i=2^{\frac{m}{2}}$
define $b_n=\frac{a_n}{v_2(a_n)}$
suppose WLOG $a_1<2^m$
suppose there is no $a_i = 2^{\frac{m}{2}}$
claim(1.1): $b_n$ is never decrease
the second operation doesn't change $b_n$
for the fisrt let $a_n=2^{k_n}b_n$ then:
$a_{n+1}=2^{2k_n}(b_n)^2+2^m=2^a(2^c (b_n)^2 +1)=2^a(b_{n+1})$
but $2^c (b_n)^2 +1 \ge (b_n)^2 +1 > b_n $
so b_n wil grow until we will have for large enough $i$ $b_i >2^m$
which is contradiction

so there exist $a_i=2^{\frac{m}{2}}$
$a_{i+1}=2^{m+1} $
$a_{i+2}=2^{m} $
$a_{i+3}=2^{m-1}=2^{\frac{m}{2}} \implies m=2$ and $a$ is a power of $2$
and we win :D
Z K Y
The post below has been deleted. Click to close.
This post has been deleted. Click here to see post.
hansu
300 posts
#8
Y by
We will check the case $m=1,2$ manually. We get that there is no solution for $m=1$ and for $m=2$, $a_1=2^t(t>0)$ works. Now we will prove that there is no such value of $a_1$ for $m>2$.

For $n=2^a b$ with $b$ odd, we will $b$ the odd part of $n$. The key observation is that the odd part of the elements of the sequence is increasing.
Proof: Suppose $a_i=2^ab$ where $b$ is odd. Then let $a_{i+1}=2^{2a}b^2+2^m$(the other case being obvious). It's easy to observe that unless $2a=m$, the odd part of $a_{i+1}$ obviously increases. If $2a=m$ we have $a_{i+1}=2^m{b^2+1}$. But observe that $v_2(b^2+1)=1$. Hence the odd part of $a_{i+1}=\frac{b^2+1}{2}\ge b$ with equality happening iff $b=1$. Thus we we will look into the case $a_i=2^\frac{m}{2}$. Thus we have $a_{i+1}=2^{m+1}, a_{i+3}=2^{m-1}, a_{i+4}=2^m(2^{m-2}+1),a_{i+5}=2^{m-1}+2, a_{i+6}=4[(2^{m-2}+1)^2+2^{m-2}]$. Thus the odd part of $a_{i+6}$ is greater than $2^m$. Thus there is no solution for $m>2$.
Z K Y
The post below has been deleted. Click to close.
This post has been deleted. Click here to see post.
square_root_of_3
78 posts
#9
Y by
Here's a sort of ugly (because of technicalities) solution, but I think the idea behind it is nice and motivated. Basically, the idea is that we'll always come back to a term in the interval $[2^{m-1}, 2^m)$, which can be written as $2^{m-1}+r$. Now we just want to prove that $\nu_2(r)<0$ at some point.

Let $m>4$ be an integer and let $2^{m-1}+r$ with $0\leqslant r <2^{m-1}$ be a term in the sequence, and suppose all terms in the sequence are integers. Then, the sequence goes as follows:
$$2^{m-1}+r \mapsto 2^m+2^{2m-2}+2^mr+r^2 \mapsto  \ldots \mapsto 2^{m-1}+2+2r+r^2/2^{m-1}.$$Now, if $2+2r+r^2/2^{m-1}<2^{m-1}$, this is our 'new' $r$. If this is not the case, then $2^{m-1}+2+2r+r^2/2^{m-1} \mapsto 2^{m-1}-2^{m-2}+1+r+r^2/2^m$, and our 'new' $r$ is $1+r+r^2/2^m-2^{m-2}$. We prove that in both cases, the $\nu_2$ decreased.

Case $1$: We will prove $\nu_2(r)>\nu_2(2+2r+r^2/2^{m-1})$. If $r=0$, the claim is obvious. If $\nu_2(r^2)<m-1$, then the right hand side isn't an integer which is a contradiction.

Since $r<2^{m-1}$, we have $\nu_2(r)<m-1$. This implies $\nu_2(2r)>\nu_2(r^2/2^{m-1}$, and if $\nu_2(2)\neq \nu_2(r^2/2^{m-1})$, we have that $\nu_2$ of the right hand side is less than or equal to $\nu_2(2)$, which is impossible.

If $\nu_2(r^2/2^{m-1})=\nu_2(2)$, then $\nu_2(r)=m/2$. In this case, $r=2^{m/2}k$ for some odd $k$. Then $2+2r+r^2/2^{m-1}=2+2^{m/2+1}k+2k^2$.

Since $2+2k^2 \equiv_8 4$, and $\nu_2(2^{m/2+1}k)>2$, we get that $\nu_2$ of the right hand side equals $2$, which is less than $m/2$. In any case, $\nu_2$ decreases and eventually it must reach something smaller than zero, which is a contradiction.

Case $2$: We will prove $\nu_2(r)>\nu_2(1+r+r^2/2^{m}-2^{m-2})$. Again, as in previous case, $m/2\leqslant \nu_2(r) \leqslant m-2$.

If $\nu_2(r)=m-2$, then $r=2^{m-2}$ since $r<2^{m-1}$. In this case, the left hand side equals $m-2$, and the right hand side equals $\nu_2(1+2^{m-4})$, which equals $0$ since $m>4$.

Suppose $\nu_2(r)<m-2$ and $\nu_2(r)>m/2$. Then $\nu_2(2^{m-2})>\nu_2(r)>\nu_2(r^{2}/2^m)>\nu_2(1)=0$, and then $\nu_2$ of the whole expression equals $0$, which is smaller than $m/2$.

The only case left is $r=2^{m/2}k$ for some odd $k$. Then the right hand side equals $\nu_2(1+2^{m/2}k+k^2-2^{m-2})$, which equals $1$ since $1+k^2\equiv_4 2$, and $\nu_2(2^{m/2}k-2^{m-2})>1$. This proves our claim.

Therefore, there are no solutions for $m>4$.

For small $m$, it's enough to check for cycles which start at a number from $[2^{m-1}, 2^m)$.

For $m=1$, we have $1\mapsto 3 \mapsto 3/2$, so there are no solutions.

For $m=2$, we have $2\mapsto 8 \mapsto 4 \mapsto 2$, so any number which eventually ends up at $2$ is a solution. These numbers are exactly all powers of $2$ except for $2^0$.

For $m=3$, we have $4 \mapsto 24 \mapsto 12 \mapsto 6 \mapsto 44 \mapsto 22 \mapsto 11 \mapsto 5.5$, $5 \mapsto 33 \mapsto 16.5$, $7 \mapsto 57 \mapsto 28.5$, so there are no solutions.

For $m=4$, all odd numbers $x$ eventually map into $x^2/2+8$, $8 \mapsto 80 \mapsto \ldots \mapsto 10 \mapsto 116 \mapsto 58 \mapsto 29 \mapsto 14.5$, $12 \mapsto 160 \mapsto \ldots \mapsto 10$, $14 \mapsto 210 \mapsto \ldots \mapsto 52.5$, so there are no solutions.
This post has been edited 1 time. Last edited by square_root_of_3, Apr 21, 2020, 12:01 PM
Z K Y
The post below has been deleted. Click to close.
This post has been deleted. Click here to see post.
Joand999
1 post
#10
Y by
thanks for your replies... still learning and i found a solution to my question. appreciating your help a lot. healthy recipes
This post has been edited 1 time. Last edited by Joand999, May 6, 2020, 4:37 PM
Reason: grammar
Z K Y
The post below has been deleted. Click to close.
This post has been deleted. Click here to see post.
VulcanForge
626 posts
#11
Y by
Define the function $f(n) = \nu_2(n)-\log_2(n)$; note that if $\frac{n}{2^{\nu_2(n)}}=k$, then $f(n) = -\log_2(k)$. I claim that $f(a_n)$ is non-increasing. If $a_n \geq 2^m$, then $f(a_{n+1}) = (\nu_2(a_n)-1)-(\log_2(a_n)-1) = f(a_n)$, as claimed. Thus we now show that whenever $a_n<2^m$ we have $f(a_n^2+2^m) \leq f(a_n)$.

Case 1: $2\nu_2(a_n)<m$. Then $\nu_2(a_{n+1})=2\nu_2(a_n)$ and we have $$f(a_{n+1})=2\nu_2(a_n)-\log_2(a_n^2+2^m) < 2\nu_2(a_n)-2\log_2(a_n) \leq \nu_2(a_n)-\log_2(a_n)$$.

Case 2: $2\nu_2(a_n)>m$. Then $\nu_2(a_{n+1})=m$ and we have $$f(a_{n+1})=m-\log_2(a_n^2+2^m)<2\nu_2(a_n)-2\log_2(a_n) \leq \nu_2(a_n)-\log_2(a_n)$$.

Case 3: $2\nu_2(a_n)=m$. Let $\frac{a_n}{\nu_2(a_n)}=b_n$; then we have $a_{n+1}=(2^{m/2}b_n)^2+2^m = 2^{m+1} \left( \frac{b_n^2+1}{2} \right)$ (note that $\frac{b_n^2+1}{2}$ is an odd integer because $b_n$ is odd). Thus $$f(a_{n+1})=-\log_2(\frac{b_n^2+1}{2}) \leq -\log_2(b_n)=f(a_n)$$.

Now observe that since $f(a_n)$ can only attain a finite number of values (assuming $a_n$ is always an integer), Cases 1 and 2 can only happen a finite number of times (because they are strict inequalities). Also note that equality occurs in Case 3 iff $b_n=1$, i.e. $a_n=2^{m/2}$. Then $a_{n+1}=2^{m+1}, a_{n+2}=2^m, a_{n+3}=2^{m-1}$. Because Cases 1 and 2 can't happen infinitely often, we must have that $2^{m-1}=2^{m/2} \Rightarrow m=2$. It can easily be checked that when $m=2$, the sequence is always an integer iff $a_1=2^k$ for some positive integer $k$. In conclusion, the answer is any even power of $2$ when $m=2$, and no solutions for other $m$.
This post has been edited 1 time. Last edited by VulcanForge, May 21, 2020, 3:16 PM
Z K Y
The post below has been deleted. Click to close.
This post has been deleted. Click here to see post.
jj_ca888
2726 posts
#12
Y by
This problem is such a typical APMO #2.

Note that at any point, we do not want $a_i$ to become odd else it will continue to be odd until it becomes $\geq 2^m$ and is then not divisible by $2$. For $m = 1$, for this reason, we see that $a_1$ must not be odd. Furthermore, if $a_1$ is even, it will be stripped of all its $2$'s and it will become odd, which is bad. Hence, $m = 1$ has no solutions for $a_1$

For $m = 2$, we also see that $a_1$ must not be odd. Let $a_1 = 2^s \cdot t$ for some odd $t$ and maximal $s \geq 1$. If $t > 4$, then we are already doomed because then we will divide out $2$'s until some term $a_i = t$ which will be forced to be divided by $2$, which is bad. If $t = 3$, then $a_1 \geq 4$ so the first move will bbe to divide out all $2$'s until some $a_i = t = 3$. Then, we have an odd term, which is bad. If $t = 1$, then note that if $s \geq 2$ we keep dividing until a term becomes $a_i = 2$ and then the next term is $a_{i+1} = 3$ and it continues to cycle\[2 \rightarrow 8 \rightarrow 4 \rightarrow 2 \rightarrow 8 \ldots\]and similarly if $s = 1$, this same cycle takes place. Hence, for $\boxed{m = 2}$, all even powers of $2$ work.

Now, consider $m > 2$. Again, $a_1$ may not be odd so let $a_1 = 2^s \cdot t$ for maximal $s \geq 1$ and odd $t$. If $a_1 \geq 2^m$, then enough $2$'s will be stripped so that some term $a_i = 2^{s'} \cdot t$ will be $< 2^m$. We start from there. See that\[a_{i+1} = 2^{2s'} \cdot t^2 + 2^m.\]When $2s' < m$, we see that $a_{i+1} = 2^{2s'} \cdot (t^2 + 2^{m - 2s'})$ so the odd part $t$ is eventually increasing. It eventually becomes larges enough to exceed $2^m$ at which point dividing by $2$ becomes impossible.

When $2s' > m$, we see that $a_{i+1} = 2^m \cdot (2^{2's - m}t^2 + 1)$ so the odd part $t$ is still eventually increasing. Similarly, it becomes large enough to a point where it exceeds $2^m$ and dividing by $2$ is again impossible.

Lastly, we consider when $2^s = m$. Note that $a_{i+1} = 2^m(t^2 + 1)$. We see that $t^2 + 1 \equiv 2 \pmod 4$ hence the odd part of this $a_{i+1}$ is therefore $\tfrac{t^2 + 1}{2}$. Note that this sequence of $t$'s becomes increasing if $t^2 + 1 > 2t$, or when $t > 1$. So when $t > 1$, we are doomed. When $t = 1$, we once again arrive at the prospect of $a_1$ being a perfect $2$th power. A quick check shows that this ends up not working, because eventually we will get a term that isnt a power of $2$:\[2^k \rightarrow 2^{k'} \rightarrow 2^{2k'} + 2^m \rightarrow 2^{m+1} \rightarrow 2^{m-1} \rightarrow \text{not power of 2}\]in the best case scenario, where a possible early break happens when $2k' \neq m$. So $m > 2$ do not work.

So indeed, our only solutions are when $m = 2$ and $a_1 = 2^k$ with $k \geq 1$. $\blacksquare$
Z K Y
The post below has been deleted. Click to close.
This post has been deleted. Click here to see post.
fukano_2
492 posts
#14
Y by
For $m = 2$ the only solution that works is $a_1 = 2^n$ for $n > 0$ and $m=1$ has no solutions. This not too hard to check.

We will prove that all $m > 2$ don't have any solutions.

Let $a_i = 2^{\nu_2(a_i)}k$ where $k$ is an odd integer for some arbitrary $i.$
If $a_i \ge 2^m,$ there must exist some $\ell \le \nu_2(a_i)$ such that $2^{m-1} \le \frac{a_i}{2^{\ell}} < 2^m \implies 2^{\nu_2(a_i)-\ell}k \in [2^{m-1}, 2^m)$ for $a_i$ to be an integer until $a_i < 2^m$ Now, if $k \in [2^{s-1}, 2^{s}),$ for some $m \ge s \ge 1.$ It is sufficient (and necessary) to let $\nu_2(a_i)-\ell = m-s \implies \nu_2(a_i) > m-s$ (1)

It is obvious that $k = \frac{a_i}{2^{\nu_2(a_i)}}$ is always increasing if $m < 2\nu_2(a_i),$ since $a_{i+1} = (a_i^2+2^m) = 2^m(2^{2\nu_2(a_i)-m}k^2+1)$ has odd part $2^{2\nu_2(a_i)-m}k^2+1,$ which is is greater than $k.$

If $m > 2\nu_2(a_i),$ then $\nu_2(2^m(2^{2\nu_2(a_i)-m}k^2+1)) = 2\nu_2(a_i)$ so the odd part is $(2^{m-2\nu_2(a_i)}k^2+1)$ which is obviously greater than $k.$

If $2\nu_2(a_i) = m,$ then $\nu_2(k^2+1) = 1 \Longleftrightarrow k^2+1 \equiv 2 \pmod{2^2}$ so the odd part will be $\frac{k^2+1}{2},$ which is greater than $k.$

To finish, note that at some point $k$ will be greater than $2^m,$ which will cause a contradiction.
This post has been edited 1 time. Last edited by fukano_2, Oct 29, 2020, 2:48 AM
Z K Y
The post below has been deleted. Click to close.
This post has been deleted. Click here to see post.
mathaddiction
308 posts
#15
Y by
If $m=2$ then $a_1=2^n$ are obviously solutions, we now show that otherwise such numbers doesn't exist.

It is easy to show that $m=1,3,4$ yields no solution, now suppose $m\geq 5$
Notice that there exists sufficiently large $n$ such that $2^{m-1}<a_n<2^m$. Therefore,
$$2^m(2^{m-2})<a_n^2+2^m<2^m(2^m)$$Therefore, we have
$$a_{n+m}=\frac{a_{n+1}}{2^{m-1}}$$In particular this implies $v_2(a_n^2+2^m)\geq m-1$ for all $2^{m-1}<a_n<2^m$. Now if $v_2(a_n^2)\neq m$, then $v_2(a_{n+1})=m$ and so
$$a_{n+m}=1$$and hence $v_2(a_{n+m}^2+2^m)=m$. Hence $v_2(a_n^2)=m$ and so $m$ is even, let $m=2k$. Then $v_2(a_{n+1})\geq 3k-1$
However, if $v_2(x)=k$ then
$$x^2\equiv 2^{m}\pmod 2^{m+1}$$and so $v_2(a_{n+1})=v_2(2^m+2^m)=m+1<3k-1$, contradiction.
This post has been edited 3 times. Last edited by mathaddiction, Feb 17, 2021, 11:16 AM
Z K Y
The post below has been deleted. Click to close.
This post has been deleted. Click here to see post.
Vitriol
113 posts
#16
Y by
The only solutions are $(m, a_1) = (2, 2^i)$ for some positive integer $i$.

Let the odd part of a number be its largest odd divisor.

Let's consider only the subsequence $(b_n)$ of $(a_n)$ with terms less than $2^m$. First we claim that the odd parts of the terms of $(b_n)$ are nondecreasing. Say $b_n=2^d \cdot o$ where $o$ is its odd part, then $b_{n+1}$ is $2^{2d} \cdot o^2 + 2^m$ divided by some power of $2$ (which doesn't affect its odd part so we can ignore it). There are $3$ cases,
  1. If $2d > m$, then $2^{2d} \cdot o^2 + 2^m = 2^m (2^{2d-m} o^2 + 1)$. $2^{2d-m} o^2 + 1$ is odd, and $2^{2d-m} o^2 + 1 > o^2 + 1 > o$.
  2. If $2d = m$, then $2^{2d} \cdot o^2 + 2^m = 2^m (o^2 + 1)$. Now all odd squares are $1$ mod $4$, so it follows that $o^2+1$ is $2$ mod $4$. Hence the odd part of $b_{n+1}$ is $(o^2+1)/2$. $(o^2+1)/2 \ge o$ rearranges to $(o-1)^2 \ge 0$, which is true, with equality at $o=1$.
  3. If $2d < m$, then $2^{2d} \cdot o^2 + 2^m = 2^{2d} (o^2 + 2^{m-2d})$. $o^2 + 2^{m-2d}$ is odd, and $o^2 + 2^{m-2d} > o^2 + 1 > o$.
The claim is proven, and furthermore we know that there is equality iff $o=1$ and $2d=m$, i.e. $b_{n+1} = 2^{m/2}$.

Clearly there are finitely many possible integer values for a term $b_n$, and at some point we must repeat a term by pigeonhole. Then since $b_{n+1}$ is uniquely determined by $b_n$, the sequence becomes periodic after this. It follows that the odd part is also eventually periodic, but it is non decreasing so it must be constant. Thus, $b_n$ is eventually always equal to $2^{m/2}$.

Clearly $m$ must be even for this to be true. Furthermore,
\[ \left(2^{m/2} \right)^2 + 2^m = 2^{m+1},\]so if $b_n = 2^{m/2}$, then $b_{n+1} = 2^{m-1}$. Thus $\frac{m}2 = m-1$, so $m=2$.

To finish, notice that for $m=2$, $a_1 = 2^i b_1$ for some nonnegative integer $i$, and we can check that
  • If $b_1=1$, $b_2 = \frac{1+4}4 = \frac54$, which is bad.
  • If $b_1=2$, $b_2 = \frac{4+4}4 = 2$, which is good.
  • If $b_1=3$, $b_2 = \frac{9+4}8 = \frac{13}8$, which is bad.
Thus $m=2$ and $a_1 = 2 \cdot 2^i$ for some nonnegative integer $i$, i.e. $2^i$ for some positive integer $i$, as desired. $\blacksquare$
This post has been edited 1 time. Last edited by Vitriol, Apr 7, 2021, 1:13 AM
Reason: \blacksquare
Z K Y
The post below has been deleted. Click to close.
This post has been deleted. Click here to see post.
L567
1184 posts
#17
Y by
The answer is it is possible only for $m = 2$ in which $a_1 = 2^k$ only, which can easily be seen to work because it goes to $2$ and then $2,8,4,2,8,4,2, \cdots$

The main idea is to look at the odd part of $a_n$. I claim this is (mostly) always increasing. Suppose $a_n < 2^m$ and say $a_n = 2^l k$. If $l \neq \frac{m}{2}$, then $a_n^2 + 2^m = 2^{2l}k^2 + 2^m$ which has odd part at least $l^2$ so its increasing.

So assume $m = 2z$ and $a_n = 2^zk$, then $a_n^2 + 2^m = 2^m(l^2 + 1)$, but $l^2 + 1 \equiv 2 \pmod 4$ so the odd part is now $\frac{l^2 + 1}{2} \ge l$ with equality iff $l = 1$. Therefore, since the odd part must eventually stabilize, we must have $a_i = 2^z$ at some point. But then see that it goes $2^z \rightarrow 2^{2z+1} \rightarrow 2^{2z} \rightarrow 2^{2z-1}$ and then goes up again, so the only way this can work is to have $z = 2z - 1 \implies z = 1$, which corresponds to $m = 2$, from which its easy to see that the only $a_1$ that work are the ones claimed above, so we are done. $\blacksquare$
Z K Y
The post below has been deleted. Click to close.
This post has been deleted. Click here to see post.
VulcanForge
626 posts
#18
Y by
Answer: only $m=2$ and $a_1 \in \{ 2, 4, 8\}$. Call the case where $a_i<2^m$ a "type 1 operation" and the case where $a_i \ge 2^m$ a "type 2 operation".

Consider the odd parts of the sequence, i.e. $a_i/\nu_2(a_i)$. Clearly this is preserved during a type 2 operation; the claim is that the odd part always increases during a type 1 operation, unless $a_i^2=2^m$. Write $a_i=2^kn<2^m$ for odd $n$.
  • If $2k>m$ then the odd part of $2^{2k}n^2+2^m$ is $2^{2k-m}n^2+1$, which is clearly greater than $n$.
  • If $2k<m$ then the odd part of $2^{2k}n^2+2^m$ is $n^2+2^{m-2k}$, which is clearly greater than $n$.
  • If $2k=m$ then the odd part of $2^{2k}n^2+2^m$ is equal to the odd part of $n^2+1$, which is $\tfrac{n^2+1}{2}$ due to the fact that $n^2 \equiv 1 \pmod{4}$. This is greater than $n$ unless $n=1$, i.e. $a_i^2=2^m$.
The $m=1,2$ cases are easily resolved by hand. Observe that if $m \ge 3$, the smallest possible result of an operation is $2^{m-1} > \sqrt{2^m}$, hence we can only get $\sqrt{2^m}$ at most once (if it's the starting value). So infinitely many of the type 1 operations increase the odd part, while type 2 operations preserve the odd part; this clearly causes the problem to shut down.
Z K Y
The post below has been deleted. Click to close.
This post has been deleted. Click here to see post.
jasperE3
11279 posts
#19
Y by
Suppose $m\ge5$.
First we claim that some $k$ satisfies $2^{m-1}\le a_k<2^m$.
If $a_1\ge2^m$ then we apply the recursion $a_{n+1}=\frac{a_n}2$ until we obtain such a $k$.
If $a_1<2^m$ we repeat this argument starting from $a_2\ge2^m$.

Then:
$$a_{k+1}=a_k^2+2^m\ge2^{2m-2}+2^m\ge2^{2m-2}$$and similarly $a_{k+1}<2^{2m-1}$. Now every subsequent term must be half of the previous one, until $a_{m+k}$. We can calculate:
$$a_{m+k}=\frac{a_{k+1}}{2^{m-1}}\Rightarrow v_2(a_{k+1})=v_2(a_{m+k})+m-1\ge m-1.$$If $2v_2(a_k)\ne m$, then:
$$v_2(a_{k+1})=\min\{v_2(a_k^2),v_2(2^m)\}=\min\{2v_2(a_k),m\}\le m$$which contradicts the previous inequality. So $v_2(a_k)=\frac m2$ for all such $k$. Now $m$ is even, so $m\ge6$. Then $v_2(a_{m+k})=\frac m2$ so $v_2(a_{k+1})=\frac{3m}2-1$. Let $a_k=2^{m/2}\cdot s$ where $s$ is odd. Then:
$$\frac{3m}2-1=v_2(a_{k+1})=v_2(a_k^2+2^m)=v_2(2^m(1+s^2))=m+v_2(1+s^2)$$so $v_2(1+s^2)=\frac m2-1$. But since $m\ge6$, $v_2(1+s^2)\ge2$, or $s^2+1\equiv0\pmod4$. This is impossble for odd $s$.

Now we deal with the cases $m\le4$.
Suppose some $a_i$ is odd. We must have $a_i<2^m$, otherwise $a_{i+1}=\frac{a_i}2$ is not an integer. But then $a_{i+1}=a_i^2+2^m>2^m$ is also odd, so $a_{i+2}$ is not an integer.
So all members of the sequence must be even.
  • $m=1$: If $a_n<2$ then that term is odd, which is impossible. But then $a_{n+1}=\frac{a_n}2$ for each $n$, which eventually is nonintegral.
  • $m=2$: The recursion becomes
    $$a_{n+1} = \begin{cases}8 & \text{if } a_n=2 \\\frac{a_n}2 &\text{if } a_n\ge4\end{cases}$$Let $a_j=2$ for the minimal $j$. Before that point, $a_{n+1}=\frac{a_n}2$ for each $n$, so $a_1$ is a power of $2$. Then $\boxed{(a_1,m)=(2^i,2)}$ for $i\ge1$ are indeed solutions, since all terms before $a_j$ are integers and we end in the loop $a_j=2\mapsto8\mapsto4\mapsto2$.
  • $m=3$: The recursion becomes
    $$a_{n+1} = \begin{cases} a_n^2+8& \text{if } a_n=2,4,6 \\\frac{a_n}2 &\text{if } a_n\ge8\end{cases}$$No member of the sequence can be $2$, $4$, or $6$ since:
    $$2\mapsto12\mapsto6\mapsto44\mapsto11$$$$4\mapsto24\mapsto12\mapsto6\mapsto44\mapsto11$$$$6\mapsto44\mapsto11$$which are all impossible. So $a_{n+1}=\frac{a_n}2$ for all $n$ which eliminates this case too.
  • $m=4$: The recursion becomes
    $$a_{n+1} = \begin{cases} a_n^2+16& \text{if } a_n=2,4,6,8,10,12,14 \\\frac{a_n}2 &\text{if } a_n\ge16\end{cases}$$Similarly, we write:
    $$2\mapsto20\mapsto10\mapsto116\mapsto58\mapsto29$$$$4\mapsto32\mapsto16\mapsto8\mapsto80\mapsto40\mapsto20\mapsto10\mapsto116\mapsto58\mapsto29$$$$6\mapsto52\mapsto26\mapsto13$$$$8\mapsto80\mapsto40\mapsto20\mapsto10\mapsto116\mapsto58\mapsto29$$$$10\mapsto116\mapsto58\mapsto29$$$$12\mapsto160\mapsto80\mapsto40\mapsto20\mapsto10\mapsto116\mapsto58\mapsto29$$$$14\mapsto212\mapsto106\mapsto53$$which are all impossible. So $a_{n+1}=\frac{a_n}2$ for all $n$ which eliminates this case too.
Z K Y
The post below has been deleted. Click to close.
This post has been deleted. Click here to see post.
Iora
194 posts
#20
Y by
Nice problem
Claim 1. All $a_i$ is even.

FTSOC, assume otherwise and let $a_i$ be odd integer for some $i$. Then either $\frac{a_i}{2}$ or $\frac{a_i^2+2^m}{2}$ is not an integer, hence contradiction. $\square$

Therefore, let $a_i= 2^a \cdot c$ where $c$ is an odd integer and $a_i>2^m$.

Claim 2. $2^{m-1} > c_i$.
Let $e$ be an integer such that $2^{e+1}>c>2^e$. Let $r$ be smallest integer such that $2^{a-r}<2^m$. Then we have

$$a_i= 2^a \cdot c <2^{a+e+1}$$. Let $a_{i+j}$ be the first time our variables goes to $<2^m$. Then $a_{i+j} = 2^{a-r-e-1} \cdot c$. But since $a_{i+j}$ is even, we have
$$a-r-e-1 \ge 1 \Rightarrow  a-r-2 \ge e \Rightarrow m-2 \ge e$$Then $2^{m-1} \ge c_i$.$ \square$.
  • If $m=1$, there are no such $c_i$, hence impossible.
  • If $m=2$, then $c_i=1$. Then any $a_1=2^x, x \ge 1$ works.
  • If $m\ge 3$,we will divide into cases.
    Case 1. $2a  \neq  m$.
    Then after some time, we will have $a_{i+j}= 2^m( 2^{2r-m} c^2-1)$ or $2^{2a}(c^2+2^{m-2a})$ then $c_{i+j}>c_i$, but it clearly increases, but $c$ is bounded above, hence contradiction.

    Case 2. $2a=m$.

    Then we have $a_{i+j}=2^m(c_i^2+1)$. Since $ c_i^2+1$ is divisible by $2$ but not $4$, we know that $c_{i+j}= \frac{c_i^2+1}{2}$. But then it is clearly increasing for $c_i>1$. If $c_i=1$, after some time we will have $a_j=2^m(2^{m-2}+1)$. since $2^{m-2}+1>1$, odd part will increase, hence contradiction.
    Therefore the only solutions are $m=2$, $a_1 =2^x, x \ge 1$ which clearly works.
Z K Y
The post below has been deleted. Click to close.
This post has been deleted. Click here to see post.
thewizard369
15 posts
#21
Y by
hi . its really nice , i get a realy easy solution for that problem but im not sure if its true
Z K Y
The post below has been deleted. Click to close.
This post has been deleted. Click here to see post.
cushie27
11 posts
#22
Y by
The answer is $a_1=2^k$ with $k \geq 1$ and $m = 2$. It is easy to check that these work and all of them lead to an eventually periodic sequence.

Assume $a_n = 2^kb$ with $b$ odd. We can also assume $a_n <2^m$ because it will become that after a few terms anyway.

We consider 3 cases: $2k<m, 2k=m, 2k>m$.

Case 1: $2k<m$
$a_{n+1}=2^{2k}b^2+2^m = 2^{2k}(b^2+2^{m-2k})$, so the odd part of $a_{n+1}$ is greater than the odd part of $a_n$.
Case 2: $2k=m$
$a_{n+1}=2^{2k}(b^2+1)$ and $b^2+1$ is $2 \pmod 4$ because $b^2+1 \equiv (2x-1)^2+1 \equiv 4x^2-4x+2 \equiv 2 \pmod 4$. Thus the odd part of $a_{n+1}$ is $\frac{b^2+1}{2} \geq b$ by AM-GM with equality iff $b=1$. If $b=1$, note that $a_k\neq a_n$ where $n \neq k$ unless $m=2$ because there is always an integer between $k$ and $m$. So we will be "moved" to Case 3 for $a_{n+1}$.
Case 3: $2k>m$
$a_{n+1} = 2^m(1+2^{2k-m}b^2)$, so the odd part of $a_{n+1} $ is clearly greater than the odd part of $a_n$. $(1+2^{2k-m}b^2>b)$.

Finally, we know that no matter what, the odd part of sequence $a$ is strictly increasing, and it will eventually reach a value greater than $2^{m-1}$. Then the sequence starts generating non-integers soon after. $\blacksquare$
This post has been edited 2 times. Last edited by cushie27, Nov 26, 2024, 3:39 AM
Z K Y
The post below has been deleted. Click to close.
This post has been deleted. Click here to see post.
pie854
243 posts
#24
Y by
For $m=2$, it's easy to check that $a_1$ works iff it is a power of two. And for $m=1,3,4,5$ we can check no such $a_1$ exists (we just need to check that $a_1<2^m$ ($m=1,3,4,5$) doesn't work). 

We claim that there exists no such sequence for $m>5$. Assume to the contrary that (wlog) $a_1<2^m$ works. Since $$\frac{2^{m-1}(a_1^2+2^m)}{2^{\lfloor \log_2(a_1^2+2^m)\rfloor}} < 2^m \leq \frac{2^{m}(a_1^2+2^m)}{2^{\lfloor \log_2(a_1^2+2^m)\rfloor}},$$there must exist a positive integer $i$ such that $a_i=\frac{2^{m-1}(a_1^2+2^m)}{2^{\lfloor \log_2(a_1^2+2^m) \rfloor }} .$

If $a_1\leq 2^{m-1}$ then $$a_1\leq 2^{m-1} \leq \frac{2^{m-1}(a_1^2+2^m)}{2^{\lfloor \log_2(a_1^2+2^m)\rfloor}}=a_i$$with equality iff $a_1=2^{m-1}$ and $a_1^2+2^m=2^t$ for some $t$. It's easy to check that this implies $m=2$, contradiction. Thus $a_1<a_i$. Repeating the same process, we get to some $a_j$ with $2^m>a_j>2^{m-1}$. So it suffices to show that $a_1\in (2^{m-1},2^m)$ doesn't work. 

Suppose $2^{m-1}<a_1<2^m$. Then, $\log_2(a_1^2+2^m)>\log_2(2^{2(m-1)})=2(m-1)$ and so $$\lfloor \log_2(a_1^2+2^m) \rfloor \geq 2m-2.$$Also $v_2(a_1^2+2^m)\leq m$; unless $a_1=2^{m/2}k$ ($k$ odd) in which case we have $$v_2(a_1^2+2^m)=m+v_2(k^2+1)=m+1.$$Thus $v_2(a_1^2+2^m)\leq m+1$. Now $$v_2(a_i)=m-1+v_2(a_1^2+2^m)-\lfloor \log_2(a_1^2+2^m) \rfloor \leq m-1+(m+1)-(2m-2)=2.$$Now note that there exists $j>i$ such that $a_j=\frac{2^{m-1}(a_i^2+2^m)}{2^{\lfloor \log_2(a_i^2+2^m) \rfloor }} .$ Then, $$v_2(a_j)=m-1+v_2(a_i^2+2^m)-\lfloor \log_2(a_i^2+2^m) \rfloor \leq m-1+(2\cdot 2)-(2m-2)=5-m<0,$$contradiction.
This post has been edited 5 times. Last edited by pie854, Mar 4, 2025, 1:29 PM
Z K Y
N Quick Reply
G
H
=
a